Can't choose any choice

This topic has expert replies
Senior | Next Rank: 100 Posts
Posts: 57
Joined: Fri Jan 22, 2010 11:25 am

Can't choose any choice

by bedazzled » Sat Jun 25, 2011 8:37 am
Pro-tect insuarnce company has recently been paying out more on car-theft claims than it expected. Cars with special anti-theft devices or alarm systems are much less likely to be stolen than are other cars. Consequently Pro-Tect , as a part of an effort to reduce its annual payouts,will offer a discount to holders of car-theft policies if their cars have antitheft devices or alrm systems.

Which of the following, if true, provides the strongest indication that the plan is likely to achieve its goal?

a) The decrease in the risk of car theft conferred by having a car alarm is greatest when only a few cars have such alarms.
b) The number of policy holders who have filed a claim in the past year is higher for Pro-Tect than for other insurance companies.
c) In one or two years, the discount that Pro-Tect is offering will amount to moret han the cost of buying certain highly effective anti-theft devices.
d) Currently, Pro-Tect cannot legally raise the premiums it charges for a given amount of insurance against car-theft.
e) The amount Pro-Tect has been paying out on car-theft claims has been greatest for some models of cars than for others.

C

User avatar
Legendary Member
Posts: 1255
Joined: Fri Nov 07, 2008 2:08 pm
Location: St. Louis
Thanked: 312 times
Followed by:90 members

by Tani » Sat Jun 25, 2011 8:49 am
This one is challenging because it changes the perspective from the company to the owner. The stimulus tells us the company is going to lower its premium for people who buy anti-theft devices.

Now we want a reason to believe the plan will work. The correct answer shifts from the company's perspective (reducing payout) to the consumer's perspective (reducing the total cost of protecting the car from theft).

For the plan to work, the car owners have to buy the anti-theft device to get the premium reduction.

(C) tell us the discount will be worth more than the cost of buying the anti-theft device. That makes it worthwhile for the owners to spend money on the device in return for the insurance savings and increases the likelihood that the plan will work.

Look at it another way. If the company offers a $20 discount per year and the anti-theft device costs $250, the consumers will not invest in the anti-theft device and the plan will fail.
Tani Wolff

Senior | Next Rank: 100 Posts
Posts: 57
Joined: Fri Jan 22, 2010 11:25 am

by bedazzled » Sat Jun 25, 2011 8:50 am
In below question, I could not figure out why C is right. This question was tricky for me. After so much effort I could rule out A D & E but was confused between B and C.
The problem with this question is that I cannot select any particular answer & had to pick up minute details to rule out options. Both B and C looked right to me. As I considered C provides information for the anti-theft devices not about the plan that would reduce annual pay-outs. I ended up marking C as right where in fact B was right.
Please explain why B is right

User avatar
Legendary Member
Posts: 1255
Joined: Fri Nov 07, 2008 2:08 pm
Location: St. Louis
Thanked: 312 times
Followed by:90 members

by Tani » Sat Jun 25, 2011 8:58 am
I don't see why B would be right. We are not concerned with Pro-Tec's rates versus competition. B also does not address the plan in any fashion. What is your source?
Tani Wolff

Senior | Next Rank: 100 Posts
Posts: 57
Joined: Fri Jan 22, 2010 11:25 am

by bedazzled » Sun Jun 26, 2011 10:52 am
Hi

In (B), I took "annual payouts as a catch" in the last line of argument & considered this as a sole cause of the problem. Deviating from the issue "Anti-theft". While in (C), they only mentioned about the devices that will get costly in 1 or 2 years.
The argument also mentioned that "Cars with special anti-theft devices or alarm systems are much less likely to be stolen than are other cars.". This seems as a bonus to me :/ and I considered something that contains information about "annual payouts" would be the right answer.

This explanation may sound awry but this happens with lots of question. Then when I look into the answer key, it's all a different story. I could not develop instinct to mark correct answer as I take the wrong catch like in here "annual payouts".

Please explain...

Bedazzled

User avatar
Legendary Member
Posts: 1255
Joined: Fri Nov 07, 2008 2:08 pm
Location: St. Louis
Thanked: 312 times
Followed by:90 members

by Tani » Mon Jun 27, 2011 8:16 am
Reducing annual payouts is the goal; the plan is to reward those who purchase anti-theft devices. You are asked to show that the goal will be achieved.

Anytime you are asked to evaluate the success of a proposal you have to ask whether it will work, whether it is feasible and whether it is practical. This proposal is very simple to implement so it is both feasible and practical. The question remains, will it work?

The plan will only achieve its goal if consumers actually take the option. Consumers will not buy the anti-theft devices and take the lower rates unless the rates save them more than the device would cost in the short term. Therefore, the answer is C. You cannot "deviate" from the anti-theft devices since they are integral to the plan you are supposed to evaluate.
Tani Wolff

Master | Next Rank: 500 Posts
Posts: 116
Joined: Tue May 31, 2011 7:52 pm
Location: Bangalore, India
Thanked: 2 times
Followed by:2 members

by Sanjay2706 » Mon Jun 27, 2011 8:24 pm
IMO C

Legendary Member
Posts: 1404
Joined: Tue May 20, 2008 6:55 pm
Thanked: 18 times
Followed by:2 members

by tanviet » Tue Jun 28, 2011 12:01 am
Why B is wrong?

Between evidence and conclusion there are many assumptions which are on the link between evidence and conclusion. The weakeners and strengtheners increase doubt of or belief in the conclusion by increasing doubt of or belief in one of those assumptions, attacking or supporting the link.

an answer choice which attack or support the conclusion outside context of evidence and which dose not attack or support the LINK is wrong. The irrelevance of this choice is tricky.

Remember, all the correct choices of most CR questions-the assumption-based questions such as Strengthening, weakening, flaw, evaluate must relevant to an assumption on the link between the evidence and conclusion.

B supports the conclusion but dose not support the link,dose not support an assumption and is wrong.

for more of this, please, read the thread "3 tricks on gmatprep" which I posted on gmatclub.com

Senior | Next Rank: 100 Posts
Posts: 57
Joined: Fri Jan 22, 2010 11:25 am

by bedazzled » Tue Jun 28, 2011 9:46 am
Thank you Tani & duongthang... I understood the point. I will definitely try to attack CR questions again with new learning.
Next time, I will ask myself "whether it will work, whether it is feasible and whether it is practical"

User avatar
Legendary Member
Posts: 540
Joined: Sat Dec 20, 2008 7:24 pm
Thanked: 37 times
Followed by:6 members

by navami » Tue Jun 28, 2011 9:18 pm
Does not D strengthen it???
This time no looking back!!!
Navami

User avatar
Legendary Member
Posts: 1255
Joined: Fri Nov 07, 2008 2:08 pm
Location: St. Louis
Thanked: 312 times
Followed by:90 members

by Tani » Wed Jun 29, 2011 7:49 am
D might tell you why they decided to lower premiums for people with anti-theft devices rather than raising premiums overall, but it doesn't tell you whether their plan will work. It doesn't address any aspect of the plan we are supposed to validate.
Tani Wolff

Newbie | Next Rank: 10 Posts
Posts: 7
Joined: Sun Oct 30, 2011 10:22 am

by prepp » Thu Oct 04, 2012 7:09 pm
Thanks for the response.
In D, if Pro-tech can't legally raise the premiums it charges, the only other plan is to offer discounts for those have anti-theft devices.

Hence the plan of offering discounts works because no other does. Is this line of reasoning correct?


Also please explain choice A:
It says that the risk is max when few cars have alarms. That would encourage others to install devices to minimize risk since they are offering discounts to do so?


Tani wrote:D might tell you why they decided to lower premiums for people with anti-theft devices rather than raising premiums overall, but it doesn't tell you whether their plan will work. It doesn't address any aspect of the plan we are supposed to validate.

Junior | Next Rank: 30 Posts
Posts: 10
Joined: Tue Nov 05, 2013 6:31 am

by tusharkhatri123 » Mon Jan 19, 2015 1:53 pm
Although I got my answer correct, but I pre-assumed something different than given in the answer choice. Please check whether this my pre-assumption is also correct:

In the premise, there is an assumption - Those cars having antitheft devices or alarm systems must not be already insured, that's why Pro-Tect Insurance Company wants to insure them in their latest approach. So, by saying that those cars were not insured before and is likely to get insured next, then it can be strengthened that the plan is likely to achieve its goal.

Thanks and Regards
Tushar